Write an expression for the sequence of operations described below. multiply 3 by t, triple the result, then raise what you have to the 8th power not simplify any part of the expression .

Answers

Answer 1

Answer: 43046721(t^8)

Step-by-step explanation:

So first, multiply 3 by t to get 3t. Then, multiply it again by 3 to get 9t. Raise it to the 8th power to get (9^8)(t^8). This gives you 43046721(t^8)


Related Questions

please help with these two questions!!

Answers

6√5 + 3√6 = 6√5 + 3√6 [cannot be simplified]

; roots do not contain any perfect squares, and the roots are not similar.

6√5(3√6) = 18√30 [can be simplified]

; although roots do not contain any perfect squares, the product rule can be applied to create a singular expression.

Solve the inequality and write the solution in interval notation:
x-6/x+5 <0

(-5, 6)

[-5, 6)

(-infinity,-5) U [6,infinity)

(-infinity,-5] U (6,infinity)

Answers

Answer:

A

Step-by-step explanation:

Firstly x cannot be -5 because the expression on th left would be undefined so it's only between choices a and c.

Create a number line with makes the expression on left 0 and undefined...so at 6 and -5 this happens.

-------(-5)--------(6)---------

Let's test the 3 intervals by choosing a value from that interval to see if all numbers from that interval will make the expression on left less than 0.

Number before -5 is -6:

(-6-6)/(-6+5)=-12/-1=12 >0 so this interval is not a part of our solution.

Number between -5 and 6 is 0:

(0-6)/(0+5)=-6/5<0 so this interval is a part of our solution

Number after 6 like 7:

(7-6)/(7+5)=1/12>0 so this interval is not a part of our solution.

The winner is everything between-5 and 6 so answer is A.

Solve each of the following:
a) x² + 4x – 77 = 0
b) x(x + 4) = -2(3x + 8)
Please show your work

Answers

Answer:

a.) x=7 or x=-11

b.) x=−2 or x=−8

Step-by-step explanation:

a)  x² + 4x – 77 = 0

Step 1: Factor left side of equation.

(x−7)(x+11)=0

Step 2: Set factors equal to 0.

x−7=0 or x+11=0

x=7 or x=−11

b.) x(x + 4) = -2(3x + 8)

Step 1: Simplify both sides of the equation.

x^2+4x=−6x−16

Step 2: Subtract -6x-16 from both sides.

x^2+4x−(−6x−16)=−6x−16−(−6x−16)

x^2+10x+16=0

Step 3: Factor left side of equation.

(x+2)(x+8)=0

Step 4: Set factors equal to 0.

x+2=0 or x+8=0

x=−2 or x=−8

Answer:

a) {-11, 7}.

b) {-8, -2}

Step-by-step explanation:

a) x^2 + 4x - 77 = 0

To factor this we need 2 numbers whose product is -77 and sum is + 4.

They are + 11 and - 7, so:

( x + 11)(x - 7) = 0

x + 11 = 0 or x - 7 = 0

x = -11,  7.

b) x(x + 4) = -2(3x + 8)

x^2 + 4x = -6x - 16

x^2 + 4x + 6x + 16 = 0

x^2 + 10x + 16 = 0

(x + 2)(x + 8) = 0

x = -8, -2.

plzzz helppp only a hour due today

Answers

Answer:

A or C

Is my best I got stuck A or C

Answer:

[tex]\text{C. about }72.05\:\mathrm{cm^2}[/tex]

Step-by-step explanation:

This is a very fun problem that requires the use of multiple concepts to solve.

Concepts/formulas used:

The measure of an inscribed angle is half the measure of the arc it formsThere are 360 degrees in a circleThe sum of the interior angles of a triangle add up to 180 degreesLaw of Sines is given by [tex]\frac{\sin A}{a}=\frac{\sin B}{b}=\frac{\sin C}{c}[/tex]  All radii of a circle are exactly half all diameters of the circleThe area of a circle with radius [tex]r[/tex] is given by [tex]A=r^2\pi[/tex]

The measure of an inscribed angle is equal to half the measure of the arc it forms. In circle Z, [tex]\angle XVY[/tex] is an inscribed angle that forms arc XY. Since XY is 40 degrees, angle XVY must be [tex]40\div 2=20^{\circ}[/tex].

Similarly, [tex]\angle VYX[/tex] is also an inscribed angle and forms arc XV. Notice how arc XY and arc XV form arc VY, which is half the circumference of the circle, since segment VY is a diameter of the circle. Since there are 360 degrees in a circle, arc VY must be 180 degrees. Therefore, we have:

[tex]\widehat{XY}+\widehat{XV}=180^{\circ},\\\widehat{XV}+40^{\circ}=180^{\circ},\\\widehat{XV}=140^{\circ}[/tex]

Now we can find the measure of angle VYX, using our knowledge that the measure of an inscribed angle is half the measure of the arc it forms.

[tex]m\angle VYX=\frac{140}{2}=70^{\circ}[/tex]

Now, we have two angles of triangle VXY. Since the sum of the interior angles of a triangle add up to 180 degrees, the third angle, [tex]\angle VXY[/tex], can be found:

[tex]\angle VXY+\angle VYX+\angle XVY=180^{\circ},\\\angle VXY+20^{\circ}+70^{\circ}=180^{\circ},\\\angle VXY+90^{\circ}=180^{\circ},\\\angle VXY=90^{\circ}[/tex]

We can now use this angle and the Law of Sines to find the length of segment VY. The Law of Sines works for any triangle and is given by [tex]\frac{\sin A}{a}=\frac{\sin B}{b}=\frac{\sin C}{c}[/tex] (the ratio of any angle and its opposite side is maintained throughout all angles of the triangle).

Since angle VXY's opposite side is VY and angle VYX's opposite side is VX, we have the following proportion:

[tex]\frac{\sin 70^{\circ}}{9}=\frac{\sin 90^{\circ}}{VY}[/tex]

Recall that [tex]\sin 90^{\circ}=1[/tex]. Cross-multiply:

[tex]9\sin 90^{\circ}=VY\sin 70^{\circ},\\9=VY\sin 70^{\circ},\\VY=\frac{9}{\sin 70^{\circ}}[/tex]

This is the diameter of the circle. By definition, all radii are half the diameter. Therefore, the radius of the circle is [tex]\frac{9}{\sin 70^{\circ}}\cdot \frac{1}{2}=\frac{9}{2\sin 70^{\circ}}[/tex].

The area of a circle with radius [tex]r[/tex] is given by [tex]A=r^2\pi[/tex]. Substitute [tex]r=\frac{9}{2\sin 70^{\circ}}[/tex] to get the area of circle Z:

[tex]A=(\frac{9}{2\sin 70^{\circ}})^2\pi,\\A\approx (4.78879997614)^2\pi,\\A\approx 22.9326052115\pi,\\A\approx \boxed{72.05\:\mathrm{cm^2}}[/tex]

Colin drove 45 minutes to the airport. He arrived 90 minutes before his flight departed, and then he spent 70 minutes in the air. Once he landed, Colin spent 20 minutes gathering his luggage, and then he drove 35 minutes to his hotel. What must be true of any expression that represents the total time that Colin spent traveling from his house to the hotel?

Answers

The answer is 50 minutes bro

Can you answer this math homework? Please!

Answers

Answer:

Height is equal = Y = 1.8 X + 3.1 = 2.3 X + 1.9

=> 2.3 X - 1.8 X = 3.1 - 1.9

=> 0.5 X = 1.2

=> x = 1.2/0.5 = 2.4

Time = 2.4 weeks

Step-by-step explanation:

Answer:

2.4

Step-by-step explanation:

B) Construct a Rhombus MARS where MR = 6.8 cm & AS = 7 cm. Write the measurement of each side of Rhombus.

Answers

Answer:

Length = 4.88

Step-by-step explanation:

Given

[tex]MR = 6.8cm[/tex]

[tex]AS = 7cm[/tex]

First, we calculate the lengths of each side of the rhombus.

Diagonals of a rhombus are bisected at right-angled.

So, the lengths (x,y) of the right-angled triangle formed are:

[tex]x = \frac{1}{2}MR = \frac{1}{2} * 6.8 = 3.4[/tex]

[tex]y = \frac{1}{2}AS = \frac{1}{2} * 7 = 3.5[/tex]

The length of the sides (z) is calculated using:

[tex]z^2 = x^2 + y^2[/tex]

[tex]z^2 = 3.4^2 + 3.5^2[/tex]

[tex]z^2 = 11.56 + 12.25[/tex]

[tex]z^2 = 23.81[/tex]

Take square roots

[tex]z = \sqrt{23.81[/tex]

[tex]z = 4.88[/tex] --- approximated

Which of the expressions below is equivalent to
4x²
12x2 - 4
X

3x²
3x
4
1
12x
1
12
A
B
D

Answers

Answer:

The choose (B)

[tex] \frac{4 {x}^{2} }{12 x^{2} - 4 } \\ \frac{4 {x}^{2} }{4(3 {x}^{2} - 1)} \\ \frac{ {x}^{2} }{3 {x}^{2} - 1} [/tex]

Multiply the binomials:

(y+2)•(y+9)

Answers

Answer:

y² + 11y + 18

Step-by-step explanation:

y² + 9y + 2y + 18

y² + 11y + 18

y^2+11y+18
explanation: y times y is y^2 y times 9 is 9y plus 2y then 2 times 9 which is 18

Solve the inequality.
-14> x-32
A. x>18
B. x< 46
C. x<-46
D. x < 18

Answers

Answer:

Option D

Step-by-step explanation:

-14 > x - 32

Add 32 to both sides;

18 > x OR x < 18

HELP?p?P?p?p?p?P?P?p?p?p?p?P?p?p?p?p?p?p?pp?p?p?P

Answers

Answer:

Yes, its a rational number.

Step-by-step explanation:

Rational numbers can be whole numbers, fractions, and decimals, and in this case it is a decimal.

Hope this helped!

Answer: yes

Step-by-step explanation:

yes 1.86 is a rational number

Solve for x. X/5-x/6=1/3 x = 10 x = 1/90 x = 1/10

Answers

Answer:

x=10

Step-by-step explanation:

I hope this will help you

10. Two planes are flying one directly behind the other. Both planes are at an alttude of 1.7 miles. The angle
of depression to the airport from the plane closer to the airport is 58. The angle of depression to the
airport from the plane farther from the airport is 37. What is the distance between the two planes to the
nearest tenth of a mile?
A 1.0
B 23 -
C 12
D Not here

Answers

It’s B trust me I have done this before

Find the values of the variables and please give the reasons​

Answers

180 ÷3 = 60 ° (angles on a straight line equals to 180 ° )

a+b+60 =180°

a+ b= 180 ° - 60° = 120°

(a+b) are 2 variables

so 120 ÷2 =60 °

therefore a and b =60 °

c +b+ 60° = 180° ( co - interior angles are supplementary angles )

c+60° +60° =180°

c +120° =180°

c =180°-120°

c=60 °

d=60° (alternate angles are equal )

or

c+b+d=180°

60° +60° + d = 180°

d=180°-120°

d=60°

(⅔)-⁴ (two over three to the power minus 4)​

I need answer asap pleaseeeee

Answers

Answer:

81/16

Step-by-step explanation:

(⅔)-⁴

81/16

= 5.0625

What is the value of y?

Answers

Answer:

C, 40 degrees

Step-by-step explanation:

All the angles of a triangle add to 180 degrees according to the Triangle Sum Theorem.

Since all angles sum to 180, we can set all the values to add to 180.

We have:

[tex]2y+y+10+50=180[/tex]

Combining like terms, we have:

[tex]3y+60=180[/tex]

Subtracting 60 from both sides gets us

[tex]3y=120[/tex]

Dividing by 3 from both sides equals

[tex]y=40[/tex]

Answer:

I think the value of y is 40

Step-by-step explanation:

Here, 2y+ y+ 10+50=180°( sum of all angles of triangle)or, 3y+ 60=180or, 3y=180-60or, 3y=120or, y= 120÷3:.y= 40

Prachi was 555 kilometers east of her home when she began driving farther east at 707070 kilometers per hour. Let f(n)f(n)f, (, n, )be Prachi's distance from her home at the beginning of the n^\text{th}n th n, start superscript, start text, t, h, end text, end superscript hour of her drive. fff is a sequence. What kind of sequence is it

Answers

Answer:

The answer is "[tex]\bold{f(n) = 70n + 5}[/tex]"

Step-by-step explanation:

The complete question is defined in the attached file Please find it.  

She started driving further east, 70 kilometers an hour, 5 km east of her home. Allow f(n) at the outset of its nth hour drive to just be Prachi's length from home.

F is a series of arithmetic.

Construct the series with an explicit formula.

[tex]\bold{f(n) = 70n + 5}[/tex]

Answer:

Arithmetic and it is f(n)=5+70(n-1)

Step-by-step explanation:

Khan Academy

PLS HELP ME ITS EASY JUST WANT TO MAKE SURE IM RIGHT Calculate the answer to the correct number of significant figures: (1.705 + 0.5067) / (0.2 * 1.243) = ______.


8.897


8.8966


8.9


9


8.90

Answers

Answer:

8.90

Step-by-step explanation:

this and the other answers are the rounded ones

What is the initial value of 34.2 x 3^x

Answers

Initial value is your y intercept, and to find that you just need to substitute 0 for x. Anything to the power of 0 is just 1. So you get 34.2(1), which means that your initial value is 34.2.

Determine the constant of variation for the direct variation given. (0, 0), (3, 12), (9, 36)
12
4
3

Answers

Answer:

4

Step-by-step explanation:

y = kx

Use point (3, 12).

12 = k * 3

k = 12/3 = 4

y = 4x

Answer: 4

Divide y by x:

12/3 = 4

36 / 9 = 4

The constant of variation is 4

Please help ASAP!!!!

Answers

Answers:  (4, 2) and (8, 2)

========================================================

Explanation:

The two points mentioned in bold are midpoints of segments AB and AC respectively.

To find the coordinates of a midpoint, you add up the x coordinates and divide by 2. Do the same with the y coordinates.

For example, points A and B are at (7,6) and (1,-2)

If we add up the x coordinates and divide by 2, then we get (7+1)/2 = 4. Do the same for the y coordinates to get (6+(-2))/2 = 2. So that's how (4,2) is the midpoint of segment AB. You'll use similar logic to find that (8,2) is the midpoint of segment AC.

A slight alternative is that once you find one midpoint is (4,2), you can draw a horizontal line until you reach (8,2). We're using the idea that the midsegment is parallel to BC which is also horizontal.

write a fraction of the probability of rolling a multiple two

Answers

Answer: 1/2

Step-by-step explanation:

there are 6 sides all together and 3 of those are multiples of two. this fraction is 3/6 but that can be simplified to 1/2

Answer:

1/2

Step-by-step explanation:

For this we will be assuming a die size of six. With that in mind, all even numbers are divisible by two. There are a total of three numbers on a six sided die, those being two, four, and six. We then put this number over the total possibilities, which would be six. It is good form to simplify, which we can. We take three out from our 3/6 to leave us with 1/2.

Sally is serving lemonade to four friends. She is serving 4/7 cup per person.

Estimate how much lemonade she needs. Then calculate exactly how much she needs. What is the difference between the estimate and actual amount?

pls help, :) ​

Answers

Answer:

oi ngl levi is hawt I like your pfp ^^

Step-by-step explanation:

my name is Riley

A student bought 84 pencils. If he sharpened 35 pencils, what is the ratio of the unsharpened pencils to the sharpened pencils?

Answers

Hello!

Sharpened => 35

Unsharpened => 84-35 = 49

49:35= 7:5

Good studies!

Answer:

7: 5

Step-by-step explanation:

unsharpened to sharpened

First we need to determine the number of unsharpened

84 - 35 =49

There are 35 sharpened

49:35

Divide each by 7

49/7 : 35/7

7: 5

What is the interquartile range of the following data set? 78,90,456,676,111,381,21

Answers

Answer

The IQR of the data set is 368.

Explanation

To find the interquartile range, you first need to find the median of the data set. Then, you find the median of the median and subtract them. This might be a little confusing but I'll walk through everything.

First, put the data set in order from least to greatest; 21 78 90 111 381 456 676. Find the median. The median of this data set is 111, since it is the middle number when the data set is ordered from least to greatest.

To find the Q1 and Q3 of the set, you have to find the median of the median.

The set right now is 21 78 90 111 381 456 676. Remove the 111 (if there were an even amount of numbers in the set, you wouldn't remove the 111 and you would just split the data set in half). Now you have two sets: 21 78 90 and 381 456 676. The median of the first set is 78 (this is the Q1) and the median of the second set is 456 (this is the Q3).

To find the interquartile range, subtract the Q1 from the Q3. 456-78=368.

Plz help me with this thank you

Answers

Answers:

One possible equation to solve is tan(x) = 4/15That solves to roughly 15 degrees

==============================================================

Explanation:

Refer to the diagram below.

The segment AB is the player's height of 6 ft.

The segment CD is the hoop's height, which is 10 ft.

There is a point E on CD such that rectangle BACE forms. This will help us form ED later.

Angle EBD is what we're after, which I'll call x.

Since the free throw line is 15 ft from the basket, this means segments EB and AC are 15 ft each.

In rectangle BACE, the side EC is opposite AB. So both of those sides are 6 ft each.

Since CD = 10 and EC = 6, this must mean ED = CD-EC = 10-6 = 4.

---------------------------------------

To summarize, we found that ED = 4 and EB = 15.

We'll focus our attention entirely on triangle EBD

We have two known legs of the triangle, specifically the opposite and adjacent sides.

So we'll use the tangent ratio.

tan(angle) = opposite/adjacent

tan(B) = ED/EB

tan(x) = 4/15 .... is the equation to solve

x = arctan(4/15) .... same as inverse tangent or [tex]\tan^{-1}[/tex]

x = 14.931417 ..... make sure to be in degree mode

x = 15 ..... rounding to the nearest whole degree

So that unknown angle in the diagram is approximately 15 degrees

CAN SOMEONE PLEASE HELP ME!!!! I NEED THIS AND I DO NOT GET IT AT ALL

Answers

Answer:

LN ≈ 14.2

Step-by-step explanation:

Given that the triangles are similar then corresponding sides are in proportion, that is

[tex]\frac{LN}{IK}[/tex] = [tex]\frac{MN}{JK}[/tex] , substitute values

[tex]\frac{LN}{41}[/tex] = [tex]\frac{9}{26}[/tex] ( cross- multiply )

26 LN = 9 × 41 = 369 ( divide both sides by 26 )

LN = [tex]\frac{369}{26}[/tex] ≈ 14.2 ( to the nearest tenth )

14.2 you’re welcome

which two consecutive integers does q lie​

Answers

Answer:

it lies between P and R because I tootk the test

write 7 pi divided by 60 in degrees

Answers

Answer:

You have reached the concluding section of our information about (7/60)×π rad to deg.

7pi/60 rad = 21°.

The radian measure is the standard unit of measure in the areas of math involving a circle like the sine and cosine function, just to name two trigonometric functions.

Using our calculator, formula and information, you have everything for changing the angle measurement at your disposal.

If you have found our post searching for 7pi/60 radian to degrees or a similar term, then you have gotten your answer as well: 7pi/60 rad ⇔ 21°.

Answer:210

Step-by-step explanation: took the test

The length of the sides of a triangle are 1115 and 20 classify the triangle

Answers

Answer:

isocles

Step-by-step explanation:

The sides are not equal

The sides are not equal so isosceles
Other Questions
Change the following sentences from active to passive voice: a) I eat oranges every morning. b) The girl hit the three balls. c) The cat drank Ada's milk. d) Mother slapped the maid. e) Stacy beat the boy last night. f) our dog, Bingo, bit the stranger last night. g) Three hefty men carried the heavy box. h) Daniel wove my sister's hair yesterday. An individual or a firm can internalize an externality by ___________. A. disputing that an externality exists. B. doubling the size of the externality. C. paying the cost of the externality. D. ignoring the externality. What is the measure of Thomas and stephani are married with four qualifying children. Their earned income is 28500. Calculate the eic using the eic formula Summarize the U.S military interactions with the Native Americans Which graph represents an exponential function? Read the following sentence and choose which word is not correct in the context of the sentence.La examen es difcil.a. Lac.difcilb.examenPlease select the best answer from the choices providedA BNEED GET POINTD Match with the correct definition. Not all terms will be used.Migration of educated professionals from a country for better pay or living conditions.Economic principles stating that a freely competitive market works for the benefit of all.Market situation in which each of a few producers affects but does not control the marketMarket situation with only one seller for a given product or serviceTwo or more consecutive quarters of decline in GDPThe amount of a commodity or service that people are ready to buy for a given price.The price determined by the movement of supply and demandThe price determined by the movement of supply and demandThe quantity of a product producers are willing to sell at a given price Recessioninflactiondemandoligopolymonopolyinvisible handsupplymarket pricebrain drain Find the slope of the line for each pair of points (-17, -5) (15, -13) Find the following sums. Please help. The legs of a right triangle have the following measurements: 5 and 10 inches. What is the length of the hypotenuse??show work. Which of the following is NOT an importance ofcell division?A. RepairB. ReproductionC. Temperature RegulationD. Growth Which characteristics describe Old Stone Age peoples? Select all that apply. Determine the value of K that will cause f(x)=Kx^2+4x-3 to intersect the line g(x)=2x-7 at one point. SHOW ALL YOUR STEPS, DON'T USE DECIMALS INSTEAD USE FRACTIONS PLEASE!!!!! What is the action of a catalyst A hotel has two types of rooms: superior rooms that cost $100 per night and basic rooms that cost $80 per night. On a given night, the hotel received $4,200 from renting 46 rooms. How many superior rooms were rented on that night? A) 20B) 21C) 26D) 30 es importante el aporte de la ciencia y tecnologia para aprovechar estas energias limpias y transformarlas de energia electrica explica Which statements describe a situation with a displacement of zero? Check all that apply.traveling south for 30 miles, then turning west and traveling another 30 milesriding on a Ferris wheel whose entrance and exit are the samewalking around the block, starting from and ending at the same houseriding on an escalator from the bottom floor to the top floorrunning exactly one lap around a racetrack The diameter of a cylinder is twice the height (h) of the cylinder. Show that the total surface area of the cylinder is:[tex]4\pi \ {h}^{2} [/tex] Simplify: (w^3)^8 * (w^5)^5